[ 3 / biz / cgl / ck / diy / fa / ic / jp / lit / sci / vr / vt ] [ index / top / reports ] [ become a patron ] [ status ]
2023-11: Warosu is now out of extended maintenance.

/sci/ - Science & Math


View post   

File: 24 KB, 500x300, 1533257707391.jpg [View same] [iqdb] [saucenao] [google]
10091015 No.10091015 [Reply] [Original]

Sleep is for the weak edition.

For book recommendations, check the sticky and/or the /sci/ wiki

For learning how to use the inboard latex, check the sticky. You can also test your latex before you post by clicking the "TEX" button in your reply box.

If your latex isn't working, it's because your adblocker is blocking it.

If you ask any question, remember that there is almost no universal notation:
>what constitutes a BAD question
If p divides |G|, show that there exists an element of order p.
>what constitutes a GOOD question
Suppose p is a prime that divides the order of a finite group G. Show that there exists an element of order p.

previous thread >>10081732

>> No.10091404

Is there an alternative to astrometry.net for doing astrometry? The site appears to have been down for a little while

>> No.10091408

Why do humans mature faster when resources are abundant when it would make more sense for them to hit their peak early in times of scarcity instead?

>> No.10091419

>>10091408
Physically I think it's pretty obvious that you need energy input in order to grow, which you don't have much of in times of low resources.
Mentally it's not really a true statement. Children in rough environments start acting like adults years earlier than their counterparts growing up in suburban Portland do.

>> No.10091472
File: 12 KB, 696x116, screen2.png [View same] [iqdb] [saucenao] [google]
10091472

who is the electrophile and who is the nucleophile here?

>> No.10091475

>>10091015
Can we πlease update the wiki?

>> No.10091489

>>10091419
>hildren in rough environments start acting like adults years earlier than their counterparts growing up in suburban Portland do
>he thinks doing chores and finding food is a counterbalance to behaving like a retarded animal

>> No.10091735
File: 68 KB, 785x251, hw7q3b.png [View same] [iqdb] [saucenao] [google]
10091735

How do I do part b here? I don't understand why the function has a maximum, as shouldn't it tend towards positive infinity as x gets closer to 0? I'm also rather confused on my professor's hint as well.

>> No.10091773

>>10091735
>shouldn't it tend towards positive infinity as x gets closer to 0?
Not really, you don't know absolutely anything about the behavior of x, other than when x tends to positive infinity. Consider [math]f(x)=e^{-x}[/math], restricted to the domain you are given. Don't assume that it is surjective.

The hint is meant for you to be able to apply Weierstrass' theorem, I assume. The idea is that you want to restrict the compact set [math][0,N][/math], because it will always have a maximum. The only thing you need to do is find an N such that the maximum in that compact interval is bigger than the supremum in [math][N, \infty)[/math], because in that case, said maximum will be an upper bound for the entire function. For that, you'll need to prove that such an interval can have a supremum for some value of N, and this is where you use the information about the limit.

>> No.10091791

Why is it that decent questions get posted here but people keep making threads for the most inane ones?

>> No.10091867

HOW TO I GET BETTER AT ABSORBING INFORMATION AAAAAAAAAAAHHHHHHHHHHHHHHHHHH

im so tired of reading shit and having it go out the other end of my head almost immediately, even trying to will myself - knowing that its important information that I want to retain - it just doesn't stick

>> No.10091887

If I missed a shot of heroin and it's still red and slightly tender in that area how many days can I wait before going to the doctor? It doesn't really feel overly warm or anything

>> No.10091889

>>10091887
Also what can I tell them if I do have to go so I don't have to admit I was shooting up drugs

>> No.10091928

>>10091889
get your life on track anon holy fuck

>> No.10092003

>>10091867
Try writing the important parts of what you are trying to remember, that usually helps me to make that information stand out, bonus if you write it from memory rather than copying it.
Also forgetting and remembering is part of the process of memorization, make sure you keep the cycle going until it is no longer necessary.

>> No.10092021

Can every system of PDE'S be derived from a Lagrangian? Or at least from the minimization of some functional?

>> No.10092070
File: 247 KB, 705x527, 1539192792877.jpg [View same] [iqdb] [saucenao] [google]
10092070

I don't understand complex numbers

>> No.10092114
File: 31 KB, 480x295, mist-forms-of-condensation.jpg [View same] [iqdb] [saucenao] [google]
10092114

It's really misty or foggy tonight and I find the air quality really refreshing. It's nice and cool, makes me feel really alive.

How would you describe the air quality scientifically and why is it that I find so pleasant?

Also why is it that stars appear clearer on nights like tonight? Cold nights, often brought on by mist or fog.

>> No.10092129

Does a laser fire at only a single wavelength?
I know there's a peak, but I don't know if it's pure.

>> No.10092211
File: 12 KB, 254x349, quat.png [View same] [iqdb] [saucenao] [google]
10092211

if quaternion multiplication is defined as in this pic, why do you even need i, j ,k for multiplication?

>> No.10092212
File: 539 KB, 1705x1056, 1528786565427.png [View same] [iqdb] [saucenao] [google]
10092212

What are some good physics lectures available in online videos?

I really loved this way to explain the Quantum Mechanic by Feynman and in general I find his lectures really good in teaching..

though they're probably outdated, is my worry..

>> No.10092215

>>10092211
Pretty notation. Working with vartesian products all the time gets tiring.

>> No.10092216

>>10092070
I don't understand mathematics, like multiplication and division is as far as my dog shit fucked brain goes

>> No.10092225

>>10092215
so the structure/notation already encodes all quternion multiplication rules?

>> No.10092236

How do I prove that xyz <= (s/3)^3 if x,y,z>0 and x+y+z=s by maximizing xy(s-x-y)? s is constant

>> No.10092250

>>10092225
Yuh. That and the cross product. ii, jj, kk=-1. ij=k, etc.

>> No.10092283

>>10091735
Consider [math]f(1)[/math] and pick [math]\epsilon>0[/math] such that [math]f(1)>\epsilon[/math]. By the limit assumption, there exists some [math]N[/math] such that [math]f(x)<\epsilon[/math] for all [math]x>N[/math]. Hence for all [math]x\in (N,\infty)[/math], [math]f(x)<\epsilon<f(1)[/math]. But now [math][0,N][/math] is compact so it attains a maximum that is larger or equal to [math]f(1)[/math].

>> No.10092317

How/why do objects go coast on forever if no force affects them?

>> No.10092538

>>10092236
Differentiate v=xy(s-x-y)=sxy-x^2y-xy^2 w.r.t x and y. Both of these will be zero at its maximum.

dv/dx=0 => sy-2xy-y^2=0 => 2xy=y(s-y) => x=(s-y)/2
dv/dy=0 => sx-x^2-2xy=0 => 2xy=x(s-x) => y=(s-x)/2

x = (s-y)/2 = (s-(s-x)/2)/2 = -s/2-x/2
=> 3x/2=s/2 => x=s/3
y = (s-x)/2 = (s-s/3)/2 = s/3

x=0 and y=0 are also solutions, but those are clearly minima.

>> No.10092593

>>10092317
Guys?

>> No.10092600

>>10092317
My brainlet answer is that's just how the laws of physics are in our universe.

>> No.10092811

Help anons, how do i go about calculating the derivative of a vector dot product with respect to one of the vectors? All I can find are answers to d/dt(a•r(t)), not d/dr(a•r). a is an arbitrary constant vector .

>> No.10092893

>>10092211
same reason we do it with regular vectors, in fact the ijk multiplication table works exactly the same as the basis vector cross product table (except with a "1" term which is trivial)
if you have a vector cross product (or a quaternion product, the rules are the same) like (5k) x (3j) it's useless to take the determinant, just do 5*3*(-i)

>> No.10092897

>>10092317
Because F=ma.

>> No.10092899

>>10092317
F=dp/dt where p is momentum (m*v)
that's the actual definition of force

if mass and velocity are constant, a=0, (total) F=0, and therefore m0v0=m1v1

>> No.10092905

>>10092317
Because this spot in space is the same as that spot in space so if it does one thing here, it must do the same thing there to be consistent. Therefore momentum is conserved.

>> No.10092932

Proofing 0 =/= 1 on an unordered field?
It seems to me like you just declare it to be 0 =/= 1 rather then that you actually proof it

>> No.10093082

>>10092932
What's the definition of 0 and 1?

>> No.10093106

>>10093082
0 is additive neutral
1 is multiplicative neutral
btw there are no other elements in my set for the field

>> No.10093109

>>10092932
It's an axiom
https://en.wikipedia.org/wiki/Field_(mathematics)#Classic_definition

Otherwise {0} is a field and you have to constantly exclude it from theorems.

>> No.10093117

How can I know if I have brain fog or retardation? my mind constantly wonders when working on anything, and whenever attempting to focus intensely, it takes a few seconds to do what feels like reeling in my mind from a pool of unrelated thoughts.

>> No.10093133
File: 133 KB, 858x332, sequencequestion.png [View same] [iqdb] [saucenao] [google]
10093133

Hello /sci/. I am in calc 2 and have been reading calculus with analytic geometry by simmons as a supplement to the course. I have been trying to work over and understand the examples but I have been stuck on this one for a while. I don't understand where the a(1/2)^k term came from. How do we know that it is greater than the terms expressed before? Thank you!

>> No.10093142

>>10093133
>I don't understand where the a(1/2)^k term came from. How do we know that it is greater than the terms expressed before?

Look up 2 lines. x/m < 1/2 so [math]\frac{x}{m+k} < ... < \frac{x}{m+1} < \frac{x}{m} < \frac{1}{2}[/math]

>> No.10093145

>>10093142
Why did they raise it the k power? I guess that is my main source of confusion.

>> No.10093151

>>10093145
because there's k terms of the form x/(m+i) in the product, all of which are bounded by 1/2.

>> No.10093153

>>10093133
>How do we know that it is greater than the terms expressed before? Thank you!
Time to break out the ol' induction
My guess is that an exponential will grow faster than an exponential divided by a factorial of the same number as the exponent at sufficient values

>> No.10093158
File: 13 KB, 530x254, Capture.png [View same] [iqdb] [saucenao] [google]
10093158

How I get to this answer? I've never seen this Lambert W function.

>> No.10093172

>20 y.o.
>in 3rd year at uni
>switched major from biology to eet because I'm shit at chemistry
>basically wasted two years since I had to take different calc classes (which is bs)
>at least resistive circuits is easy peasy so far, also pretty cool
How bad is it taking more than 4 years to get a bachelors? I'm gonna need at least 5 since I was a retard and did worthless shit for two years.

>> No.10093173

>>10093151
>>10093151
>>10093151
>>10093151
Oooooh wow thank you so much. So because x/m is bounded by 1/2, x/m+1, x/m+2, ... are all bounded by 1/2. Therefore x^m/m! *(1/2)^k has to be greater than the previous terms. Ty.

>> No.10093179

>>10093172
You're young. If you like and can afford it, do it.

>> No.10093182

>>10093151
Also, the author states "An increasing sequence converges if and only if it is bounded". Doesn't this seem like circular reasoning? Isn't the upper bound of a sequence the value that the sequence converges to?

>> No.10093208
File: 2.36 MB, 1824x2423, LambertW_UWO_Poster.png [View same] [iqdb] [saucenao] [google]
10093208

>>10093158
Let k=(-x + xlog(x) + c1)/-2 because I'm not writing that out repeatedly

1/y + log(y)/y = k

1 + log(y) = k*y

k*y - log(y) = 1

e^{k*y} / y = e

1/e = e^{-k*y} * y

Let z = -k*y

-k/e = e^{z} * z

W(-k/e) = z = -k*y

y = -W(-k/e) / k

>> No.10093211
File: 30 KB, 550x235, ET spectrum.gif [View same] [iqdb] [saucenao] [google]
10093211

>>10093172
>eet
>electrical engineering TECHNOLOGY

That's a scam degree and not actual EE.

>> No.10093214

>>10093182
No. The statement just means that if a sequence is unbounded, it can't converge.
>Isn't the upper bound of a sequence the value that the sequence converges to?
Why do you assume that it is an upper bound? In your original image, the limit is zero, which is obviously not an upper bound of the sequence.

>> No.10093218

>>10093172
I have to take min 5, probably 5.5, since I switched to physics in my sophomore year. Its not too bad if you can afford it

>> No.10093220

>>10093182
0 1/2 2/3 3/4 4/5 ... is bounded above by 2 and 3 and 4 and 5 and ...

>> No.10093228

>>10091735
f(0) is defined so it can't blow up there. The domain is [0, and not (0, for exactly that reason.

>> No.10093231

>>10091475
Which one? The google site or the wikia? The wikia does get updated and if there's something you want to add, add it...
https://4chan-science.wikia.com/wiki//sci/_Wiki

>> No.10093234

>>10091867
Stop watching porn.

>> No.10093245

>>10091867
gotta apply it or practice it otherwise you'll lose it
if you're trying raw memorization then you're going to have to keep up with it
but don't expect to ever be able to read some historical book and retain all that information, you'll lose most if it no matter what

>>10093234
meme

>> No.10093259

Really dumb question on boolean algebra
how do I simplify AC + A'B'C?
The only thing I can think of is C(A+A'B')
Also, how would I map AC+A'B'C' in a truth table or kmap? Am I supposed to make 2 truth tables/ Kmaps because there are 2 inputs for AC and 3 inputs for A'B'C?

>> No.10093276

>>10093259
>how would i map it in a truth table
A B C RESULT
true false true = true
etc.

would answer your main question but i don't want to

>> No.10093287

>>10093276
>true false true = true
So you're saying AC = 101 where 0 is B? So AB'C = AC?

>> No.10093293

>>10093287
No. If you write a truth table with 8 entries, both of the entries for which A and C are true will satisfy the expression.

>> No.10093296

>>10093259
>The only thing I can think of is C(A+A'B')

A = A(1) = A(1+B') = A + AB'

C(A+A'B') = C(A+AB'+A'B') = C(A+(A+A')B') = C(A+B')

>> No.10093299

>>10093245
>meme

>t. pornjew

>> No.10093302

>>10092129
No, it isn't pure per se in a single wavelength, but it's a narrow frame around the peak

>> No.10093306

>>10093293
Right. I was just over thinking this lmao

>>10093296
Thanks. Was also able to verify with a kmap

>> No.10093401

>>10093208
Thank you very much. Could you explain how you got from 1/e = e^{-k*y} * y to -k/e = e^{z} * z, please?

>> No.10093563

>>10093299
i don't think you should watch porn or masturbate but it's a big meme to think you'll gain superpowers by not masturbating

>> No.10093594
File: 339 KB, 882x768, Screenshot 2018-10-24 at 7.39.22 PM.png [View same] [iqdb] [saucenao] [google]
10093594

How does he get that x goes from 0 to (1-y)?

>> No.10093607

>>10093594
See the line between the two 1s in the diagram? That's the graph of y = 1-x. Now isolate x.

>> No.10093633

How does the hidden layer of neural networks work?

I'm trying to build my own basic one that imitates the sin function, but I'm having trouble understanding the hidden layer.
There's the input then the hidden layers then the output. The hidden layers do all that math, but what kind of math does it perform, is that something I decide or am I missing something?

>> No.10093652
File: 16 KB, 800x600, 1540423410552.png [View same] [iqdb] [saucenao] [google]
10093652

I had made a thread, but it would probably have been a better idea to ask here first.
Does/would the expansion of space partially cause/add to diffraction of a coherent light beam, and conversely would a region of contracting space compress a low-spread light beam to a smaller diameter?

>> No.10093676

>>10093633
hidden layers add more linear functions to it
if you have no activation function (don't do this) then you generally get output=weight1*input+weight2*input+weight3*input or something similar
if you have 2 inputs, it's output=weight1*(input1+input2)+...
but with the activation function it changes it, instead you have output=S(S(w1i1)+S(w1i2)+...) which adds more complexity to it
the next hidden layer, if there is a next one, complicates it further with S(W1*S(w1i1)) where W1 is the second layer weight

the activation function is basically a way to normalize the output
using a nonlinear activation function creates an infinite series using these weights (just as sin(x) is an infinite series), this means that nonlinear activation functions are all universal function approximate, as in they can approximate any function

the hidden layer just adds complexity, that's just about it

>> No.10093708

>>10093214
Sorry I meant won't one of the bounds of the sequence always be the value which the sequence converges to? I didn't mean for it to specifically be the upper bound.

>> No.10093725

>>10093652
>expansion of space cause diffraction
sure, each point of space is being stretched out so you essentially change the frequency an extremely small amount
you get a redshift from expansion and contraction (blueshift) of space

>> No.10093745

>>10093725
I knew about the red and blue shift, but I was wondering more about diffraction
Could one measure how fast space is expanding or contracting locally by exploiting diffraction? If you shot a laser beam back and forth through a narrow aparture, increased diffraction would cause increased decay, right?

>> No.10093772

>>10093676
so the hidden layer is just the outputs of the previous layer multiplied by weights and outputted to the next layer?
The activation function puts these outputs into a set range?

I'm not sure what you mean by non linear activation function.

>> No.10093780

>>10093745
yeah i suppose diffraction could happen as a result of it
you do know it's like 68 km/s for each megaparsec of distance, or 6.8m/s per 3*10^18 meters from the observer
that's 2.2*10^-18 m/s per meter apart
probably did one or two unit conversions wrong somewhere but regardless, you'd need such a massive testing site that extraneous error would come about for sure

>> No.10093803

If I have an associates degree and a bachelors degree should I list both on my resume?

>> No.10093806

>>10093772
nonlinear activation function would be like arctan(x) or the sigmoid function

didn't originally say it right, here's what the output would be for 2 inputs and 3 hidden layer nodes, h1 being the hidden layer weight, S() being the activation function
output=S(h1*S(w1i1)+h2*S(w2i1)+h3*S(w3i1)+h1*S(w1i2)+...)

activation function adds complexity, because it's a non linear function, you're able to approximate any function with sufficient data and training

the hidden layer drives the whole thing, the point of it is to add weights for each of those S(w1i1) terms
if you didn't have it, you'd just end up with output=S(S(i1*w1)+S(i2*w2)), which is 4 times less variables, and you'll get a piss poor estimate for most functions because of how linear it is

>> No.10093816

>>10093401
Multiply both sides by -k, make substitution

>> No.10093819

>>10093780
So the diffraction due to expansion should probably be inconsequential

>> No.10093822

>>10093819
not inconsequential, it surely matters at the megaparsec level
but absolutely fucking useless for measurements

>> No.10093828

>>10093633
nn are a meme, they're just obfuscated curve fits.

>> No.10093834

>>10091735
Define continuity wrt the chaotic topology and state that the proof is trivial. When the professor calls you out on it, just tell him that the question was not well-defined and it's not your fault he didn't specify the open ball topology.
Bonus points: when he makes you redo it, do it with the discrete topology.

>> No.10093848

>>10092129
You can never shoot light at a single frequency. How narrow the frequency bandwidth is (fwhm) is measure of how good your laser is, though.

>> No.10093860
File: 40 KB, 652x569, noicee.jpg [View same] [iqdb] [saucenao] [google]
10093860

>>10093816
Thanks again. You helped a lot!

>> No.10093880

>>10093822
Thanks

>> No.10093897

>>10093806
I think I understand the hidden layers now. But how does an activation function make sense of a value and normalize it? If I input a 5 as my input and then my network spits out a -1 after being pass through the activation function does the activation function just take the value and pass it through as an input? If it does this how does it not mess up output for a really big number?

>> No.10093923

>>10093897
>how does it make sense of it and normalize it
large values get scaled down closer to 1 for something like the sigmoid function, or anything with a (-1,1) range
if you need it to parse large values differently, you could use something like log(1+e^x) or x*sigmoid(x)

as far as your input question, no, the value you gave as an input is the input
the hidden layer values are S(wxix)
those hidden layers add n different values because each tree has its own weight, the output is determined from output weights times those hidden values (with the activation function applied to it all)
if you have m inputs and n hidden nodes, you have n*m different values as "inputs" all dependent on the m inputs

>> No.10093966

>>10093923
I get it now. I got a couple more questions. How do I choose what my inputs should be ,how many hidden layer I need, or what activation function to use? For a network imitating a sin function you would only need one input the input of the function,the activation function would be a function that normalizes all values between a range of -1 to 1, right? But how many hidden layers should there be and how many nodes for a layer?

>> No.10093975

>>10093966
>how do i choose my inputs
depends vastly between what you're doing
you said something about a sine curve, so i'd pick like every third degree or something (0, 3, 6, ... 360) maybe 5 it's up to you

>hidden layers
i could write for days about this one
if you have a shit ton of data, go for 2-3

>nodes
make them symmetrical, each layer having the same amount of nodes
rule of thumb is inputs+outputs, so if you have 2 inputs and 1 output, each layer should have 3 nodes
>activation function
choose dependent on range, reminder your highest output you could get is the range of that function
you can always multiply the function itself by a constant to scale it to 2 or something if you need to
sigmoid works just fine for most applications

>weights (didn't ask but it's important)
randomly setting them on a gaussian distribution works
he-et-al or xavier works well as well
sometimes you want to make sure your gradient descent isn't just getting stuck at like .25 and won't go lower because the learning algorithm is increasing one weight and decreasing another the same amount, so you have to look out for that kind of thing when making your algorithm

when you do back propogation that's when you compare the nn output to the expected btw

>> No.10094023

>>10093975
You've lost me here. I'm not sure what you mean by gradient descent. What is the learning algorithm, is it the neural network? If I don't do back propagation to compare the output of the net with the expected, how does the net correct itself if this where the learning algorithm comes in?

>> No.10094029

>>10094023
you uh might want to look up what forward and backward propogation are
i really don't want to type out the math involved

>> No.10094043

>>10094029
Alright. Thanks for taking the time to explain all of this to me. One more question, right now I'm using some old books from my uni's library to learn about neural networks, do you know of any learning resources you could recommend?

>> No.10094054

>>10094043
andrew ng's course is okay for an introduction to it all (covers much more than what i've talked about here, but still pretty basic)
past that you really have to find resources on your own for whatever you're trying to do

>> No.10094059

Why is it that the superpartner particles would have their spin off by 1/2? Shouldn't they have the same spin?

>> No.10094063

>>10094054
thanks anon

>> No.10094076
File: 43 KB, 630x630, 1e3.jpg [View same] [iqdb] [saucenao] [google]
10094076

Here's the question:
We have the temp of a room: u(t) which is related to the heater power P, heat sources in the room S, and the temperature of the room itself which can bleed out.

\frac{\mathrm{d} u}{\mathrm{d} t}=aP+S-bu

P=P_{0}-K\int (u-u_{0})
\frac{\mathrm{d} P}{\mathrm{d} t}=-K(u-u_{0})

Show steady state is u0 and this doesnt depend on any system parameters like feedback strength K, additional heat sources, or thermal coupling to outside or heater.

Just ending up in circles du/dt=0 so that mean u=u0 then u=u0 so dp/dt=0 so du/dt=0. Cant simply show u=u0 is the steady state temp.

>> No.10094111

If I perform a Laplace transform on a piecewise function f(t) defined over intervals in t, what are the intervals in s for F(s)? Is such a thing even meaningful? My text doesn't properly discuss the frequency domain.

>> No.10094127

>>10094111
You have to split the integral at the discontinuities. The resultant F(s) will not have any intervals.

One thing to note is that your piecewise function has to be defined for all positive reals.

>> No.10094128

>>10094111
There aren't any. Just break up the integral into the intervals and sum. Just like integrating any piece-wise function.

>> No.10094139

>>10091015
I am about to finish my degree in CS. I don’t have much EE experience but I want to build robots. Anyone got any good books?

>> No.10094144

>>10094139
>I am about to finish my degree in CS

I am so sorry.

>> No.10094156

>>10094144
Fym sorry? I already have a job making bank dude.

>> No.10094389

Why do metals reflect visible light but let short wavelength radiation through?

>> No.10094450

suppose player A has 0.7 chance to win, B 0.3.
The score is 0-1, player B is winning.
First one to 3 is the victor.
What is the probability of player A winning?

is this right?
[math]0.7^3 + 0.3 * 0.7^3 * {4\choose3}[/math]
~0.75

>> No.10094483

Asking here too

>>10094474
>>10094474

>> No.10094491

>>10094450
Player A has 0.7 chance to win.

>> No.10094528

>>10094450
https://onlinegdb.com/SyrSTzyn7

No, you counted an extra A wins wins wins losses. P = .7^3 + .3*.7^3 * ((4 choose 3) -1) = .6517

>> No.10094535

>>10094528
thanks.

>> No.10094542

>>10094483
How much stuff is there in the flashcards? I think your amount of reviews sounds reasonable.

>> No.10094546

>>10094542
Most of the new ones are answers of a few words, while maybe half of the reviews are diagrams with 10+ structures on them. I've been doing the deck for most of the semester but over half of those cards were added in the last two weeks.

>> No.10094566

>>10092212
Lads?

I've binge watched ton of Feynman videos in Youtube and love them. He is such a great teacher.

>> No.10094600

>>10093563
Now that's a meme.

>> No.10094669

>>10094450
> is this right?
Nope. Out of 4 rounds, A must win 3 or 4, so it's
C(4,3)*P^3*(1-P)+C(4,4)*P^4
= 4*0.7^3*0.3 + 0.7^4 = 0.6517

>> No.10094700

I'm aa generally stupid fellow. I have bad math brain.
But I would like to learn about probability. It interests me.
What are some good books or youtube videos? is Taleb good source? He has made me interested in probability. But my shit for brains can't really tell if I'm reading smarts or stupids

>> No.10094702

>>10094700
>is Taleb a good source
No, not really.
>inb4 t. Economist
True enough, but he isn't really a mathematician.

>> No.10094708

>>10094702
I'll try to look up university lectures in YT on probability then, thanks.

I learned Calculus by finding out the syllabus of Yale to its math courses (outdated, but still)

>> No.10094910

>>10091472
The two groups of interest are the O and the CH3CH2I, I’m sure you can figure out the rest

>> No.10094956

Help, I'm having a brain fart. If I have a set S that is a power set of a set, and R = [7] the set containing [1,2,3,...,7], I CAN make a function f: S -> R, right?

Like an element of f would be like ({1,2,3}, 1) or something.

>> No.10095039

Given a solenoid through which electric current is flowing, and a ferromagnetic body free to move along the solenoid's central axis, I must compute the force acting on the body as a function of its distance from the solenoid's center of mass.

https://www.researchgate.net/post/How_to_find_force_acting_on_ferromagnetic_material_when_it_is_placed_in_external_magnetic_field

What measurements can I take to determine the ferromagnetic body's magnetic moment?

>> No.10095044

>>10095039
https://en.wikipedia.org/wiki/Magnetometer

>> No.10095050

I dropped maths years ago in my countries equivalent of highschool and now find myself woefully ill equipped for a PHD program i have started recently. Where should i look for resources to teach myself the foundations needed for stats and the likes.

>> No.10095072
File: 14 KB, 472x56, IMG-20181025-WA0005_20181025134903853.jpg [View same] [iqdb] [saucenao] [google]
10095072

Guys, shouldn't dividing everything by 2 cancel out the 2 in pi/2? Why is it multiplying the denominator?

>> No.10095118

>>10095050
Check out your unis syllabus for math subjects..?

>> No.10095252

>>10095072
> Why is half of 0.5 not one?

>> No.10095272

>>10095252
Ooooohhh, jesus fuck I hate being retarded.

>> No.10095375
File: 42 KB, 787x509, circuit.png [View same] [iqdb] [saucenao] [google]
10095375

Tested this circuit in lab today. The 6 Ohm resistors are actually speakers (didn't know how to properly model them).

For the life of me, I can't seem to fully understand what function the capacitor serves. I tested it on a frequency ranging from 1 Hz to 10kHz, and at around 1.3kHz and 1.5 Hz there was a jump in voltage amplitude dropped across the capacitor. Between 1.5kHz and 1.9kHz it the voltage amplitude went back to where it was before and stayed there.

I just started doing electronics (as you can obviously tell) but increasing the frequency will decrease the impedance in the capacitor, right? If that's true then as I increase the frequency more current should pass through the capacitor as compared to the speaker and since the capacitor has no (or negligible) resistance then the voltage dropped across the parallel combination should keep dropping as I increase the frequency. But it doesn't really. If I jack up the frequency then the voltage drop actually increases as I start moving into the MHz territory - is this because of parasitic effects in the wires themselves?

Sorry for all the noob questions.

>> No.10095378

>>10095050
How to Solve It: A New Aspect of Mathematical Method by Polya
Basic Mathematics by Serge Lang
Book of Proof by Hammack
Methods of Mathematics Applied to Calculus, Probability, and Statistics by Richard Hamming
The Art Of Probability: For Scientists and Engineers by Richard Hamming

https://4chan-science.wikia.com/wiki/Mathematics

>> No.10095380

>>10095378
>>10095378
>The Art Of Probability: For Scientists and Engineers by Richard Hamming

I'm mostly just a stupid faggot interested in what probability means in general, is this good? I guess its better than nothing anyway.

Thanks, should of checked wiki.. (Not the guy you reply to)

>> No.10095426

>>10095375
Are the values in your diagram accurate?
What you have built is a low pass for the speaker on the far right, as I'm sure you know. The cutoff frequency ([math]\frac{1}{2\pi R C}[/math]) for the dimensions you have indicated is 33.8 MHz.. so I'm not quite sure what you have observed down there in the kilohertz range. Hope that helps.

>> No.10095435

>>10091015
https://en.wikipedia.org/wiki/Zero-energy_universe
this is legitness right?

>> No.10095439

Tl;dr everybody in sci were raped when they were young and didn't believe that their rapists would get to heaven before them
Einstein is a nigger
#Yolo

>> No.10095458

>>10095375
The speak is an electromagnet, which is a coil of wire, which is an inductor, which cancels out the capacitor with resonance.

Z = 1/Ciω || Liω + 6 = 1/(1/(Liω + 6)+Ciω)

>> No.10095468

>>10095380
Yes.

>> No.10095513

>>10095426

Greatly appreciate the help. I asked 3 different lab demonstrators about the quirk and all said that's how it was supposed to be but none of them actually managed to explain why the voltage went up and then went back down again. Could it be just resonance with parasitic effects or something? It seemed abit severe though because the amplitude rose by about 33%.

Also, any ideas on why they chose such a tiny capacitance to demonstrate the effects of low pass filters when we were only instructed to operate wayyyy lower than the cutoff frequency? The whole experiment seemed like it was poorly designed - at least if the intent was to educate fools like me.

>>10095458

Ah, thanks for that important insight.

>> No.10095533
File: 8 KB, 504x120, Limit.gif [View same] [iqdb] [saucenao] [google]
10095533

Hi! Could anyone explain to the dumb me why this is true?

>> No.10095539

>>10095533
log_b(x)=ln(x)/ln(b)

log_2(x+1)/log_3(x+1) = (ln(x+1)/ln(2)) * (ln(3)/ln(x+1)) = ln(3)/ln(2)

yes it's a constant

>> No.10095542

>>10095533
Because [math] \log_a b = \frac{1}{\log_b a} [/math].

>> No.10095548

>>10095533
What is change of base in logarithms.

>> No.10095565

>>10095533
log(a)/log(b) = log base b(a)

>> No.10095593

>>10095513
But what was the setup about? I'm sure you received some sort of instructional material. Post it, I'm interested.

>>10095458
Also, from what I could find about the typical inductance of a speaker that size, the resonance frequency would still be around 10 MHz.

>> No.10095603
File: 140 KB, 985x1500, 1526981348052.jpg [View same] [iqdb] [saucenao] [google]
10095603

how do I get the last drops out of a glass eye dropper vial without anything else i.e no syringe or pouring it out into a second container?

>> No.10095604

>>10095539
>>10095542
>>10095548
>>10095565
Thanks, now I feel slightly less dumb.

>> No.10095607

>>10095603
smash the glass

>> No.10095631
File: 15 KB, 620x420, iu.jpg [View same] [iqdb] [saucenao] [google]
10095631

>>10095607
> shoot the glass

>> No.10095633

Would someone please posts all the math memecharts available?

thanks

>> No.10095642
File: 81 KB, 847x873, assdfae.png [View same] [iqdb] [saucenao] [google]
10095642

>>10095593

These were the instructions. Sorry for the late reply.

>> No.10095650
File: 42 KB, 827x411, retasdfav.png [View same] [iqdb] [saucenao] [google]
10095650

>>10095642
>>10095593

And this was the circuit diagram.

The thing itself was about using LEDs to sync with the frequency of sound but obviously at frequencies where the blinking was visible (f<50 or so), the sound wasn't audible.

>> No.10095654

>>10095650

Forgot to mention that there is a mistake in the diagram. There should be a 470 Ohm resistor in series with the first speaker to prevent all the current from just travelling around that loop instead of through the rest of the circuit.

>> No.10095673

>>10095593
>Also, from what I could find about the typical inductance of a speaker that size, the resonance frequency would still be around 10 MHz.

Which is why Z keeps increases. Passed it, it starts decreasing like you expected.

>> No.10095676

>>10095642
>>10095650
>>10095654
Thanks for providing that. Everything reads as if the goal was to make the low pass cutoff audible -- but it is a weirdly designed setup, in my opinion. Interesting nonetheless..
Anyway, for the cutoff to fall within the audible part of the spectrum the capacitor should be around 10 to 20 µF. Maybe you can try that next time you get the chance.

What kind of course environment is this from?

>> No.10095690

How do I get better at problem solving?

>> No.10095699

>>10095676

First lab I'm doing in a basic experimental skills module for a Mechatronics course.

Changing the capacitor definitely sounds like it would make for a much more interesting experiment. Fortunately the next experiment we'll be doing in a couple weeks will cover charging and discharging in an RC circuit so I'll get more time to let this sink in.

>> No.10095708

>>10095690
>How to Solve It: A New Aspect of Mathematical Method by Polya
https://4chan-science.wikia.com/wiki/Mathematics#Problem_Solving_and_Heuristics

>> No.10095862

what the HELL is a "moment generating function" and what does """"""moment"""""" mean? why is it in physics and probability and other shit? why the word "moment" to describe whatever it is

>> No.10096045

/sci/ I have an interest in learning about the wildlife of my state. aquatic, fauna, wildlife, insects, arachnids, etc.

Is there some way to learning what wildlife biology books my local colleges use to get ideas of whats good?

Colorado btw. I have previously taken an elective wildlife biology 101 in another state and found it super interesting, but college credits are fucking stupid expensive.

>> No.10096049

>>10095862
It deals with the shape of the distribution of something.

https://en.wikipedia.org/wiki/Moment_(mathematics)

>> No.10096084

Does anyone know a free site where I can generate wind roses using my own data? The only ones I can find only allow you to select from their databases.

>> No.10096089

>>10096084
Just look at the options on polar graphs in your favorite graphing software.

>> No.10096173
File: 50 KB, 704x188, Untitled.png [View same] [iqdb] [saucenao] [google]
10096173

Someone explain this to a brainlet, I could answer every question except this one.

>> No.10096206

>>10096173
what have you tried?
are you at least able to find one angle?

>> No.10096223

>>10096206
I rearranged the the formula for Range to get Theta. Am I on the right lines?

>> No.10096249

>>10096223
yep, I think I know where you're getting caught up.
Consider the inverse sine function, what is it's output range? what is the output range of the formula you found? and what output range do we want?

if you are having trouble answering the last question think about how we would want to orientate the cannon, clearly we don't want to be aiming at the ground, as well as we don't want to be aiming backwards either.

>> No.10096291
File: 76 KB, 1215x445, rotation.png [View same] [iqdb] [saucenao] [google]
10096291

Why the ethers obtained by both routes have optimally opposite rotations?

>> No.10096361

https://physics.stackexchange.com/questions/334802/ground-state-energy-in-terms-of-partition-function?rq=1 What in the name of fuck is this garbage? I had this on my exam and it's obviously bullshit. This is beyond heuristics this is plain wrong. Take ln(n) for a quick counterexample.

>> No.10096392

>>10096173
At 45° you get the maximum range. Any short range and you must fire above or below 45° leaving to a slower and faster arrival respectively.

[math]
v \sin(\theta) -g*t = -v \sin(\theta) \\
t = 2 v \sin(\theta) / g \\
x = v \cos(\theta) * 2 \sin(\theta) / g \\
d = v^2 \sin(2*\theta) / g \\
\theta = \arcsin \left ( \frac{dg}{v^2} \right ) / 2 = 15.359...° , 74.64...° \\
t_{slow} = 2 v \sin(74.64°) / g = 127s \\
t_{fast} = 2 v \sin(15.36°) / g = 35s
[/math]

>> No.10096400
File: 1.04 MB, 1080x1080, a1d6847073ad8fa4c8ff0b56dcd7826e.jpg [View same] [iqdb] [saucenao] [google]
10096400

So i have a feedback control problem im stuck on. I have a controller with form K(s+a)^2/(s+b)^2. It tells me to find parameters that make ss error for a step zero, and have less than 10 percent over shoot. Tells me to use matlab. How the fuck do i do this?

>> No.10096729
File: 14 KB, 387x357, Captura de ecrã de 2018-10-26 11-10-56.png [View same] [iqdb] [saucenao] [google]
10096729

Why is this mathematically not allowed (it was part of a document for bringing new Physics students up to speed/back in shape Math-wise, the document said it works fine, but isn't allowed in strict Maths)

>> No.10096738
File: 99 KB, 768x959, d7855b5672462c701e17b28d202985d2.jpg [View same] [iqdb] [saucenao] [google]
10096738

When exactly can I do analysis over a field? What are the basic requirements? I know you need a notion of closeness from Topology (can't remember what it is exactly though.)

Also if I can do Calculus over any Field are there an infinite number of "Calculi" or "Analysis's" that one can study? Specifically from Cayley-Dickinson Construction.

i.e Quaternion Analysis, Octinion Analysis, Septinion Analysis etc?

>> No.10096777

>>10096729
Because dx isn't anything.

>> No.10096786

>>10096729
Because the book was written before the 1960s and Abraham Robinson making nonstandard analysis a thing.

>> No.10096789

>>10096738
>When exactly can I do analysis over a field

When it's complete so you know limits exist.

https://en.wikipedia.org/wiki/Complete_field

>> No.10096837

>>10096249
>>10096392
Thank you both, I've answered both questions and I understand now.

>> No.10096839

>>10092216
Even if you can understand up to multiplication and division, I assume you know addition and subtraction, that still allows for algebra to be taught to you, such as plugging in numbers in place of a variable, or using addition, subtraction, multiplication and division to solve for a variable, leading you to higher math education paths! you aren’t hopeless yet anon, you just need a good teacher and some time!

>> No.10096914
File: 3.13 MB, 800x5544, A Guide.png [View same] [iqdb] [saucenao] [google]
10096914

>>10095633

>> No.10096923

>>10096729
literally just, in the third step, dont multiply by dx
then in the fourth step, integrate both sides with respect to x. use the substitution on the left side "y=x" to eliminate the dy/dx, and you get a completely rigorous mathematical proof. in other courses, you just skip that step

>> No.10096931

>>10096738
Metric spaces are the generalisation of where one can do analysis.
The most interesting spaces are the ones that are complete in the sense of Cauchy sequences, that basically says "there are no missing points". Some examples are: Q-adics, or on some smooth manifolds.

Btw, Quaternions and Octonions are not a field since they are not commutative (they are still a metric space though, since they have a norm. Sedenions don't have a norm so perhaps not)

>> No.10096976
File: 11 KB, 802x75, file.png [View same] [iqdb] [saucenao] [google]
10096976

Can someone explain to me how this is discretised from Navier-Stokes? any papers would be useful thank you

"A three-step Runge–Kutta scheme is used for the convective terms while the viscous terms are treated by
the Crank–Nicholson method, leading to overall formal second-order temporal accuracy."

>> No.10097035

Has there been any interesting cases of human experimentation and the result of it being beneficial or just useless?

>> No.10097048

>>10097035
stanford prison experiment

>> No.10097051
File: 150 KB, 1920x1080, 1530045152532.jpg [View same] [iqdb] [saucenao] [google]
10097051

>>10096839
I have no teacher other than myself. I failed to get into any university in 10 years that I tried and applied.

I wish Jeff Bezos would turn me into fucking biodiesel.

>> No.10097052

>>10097048
Not replicable.
Psychology is a pseudo-science used to justify a lot of absolutely horrid shit down the people's throats.

See: diversity oaths in modern academia.

>> No.10097148

>>10097051
How is that even possible. Were u poor as fuck with a bad family life and got shit grades in highschool?

>> No.10097150

>>10097148
Yes, yes and yes.

>> No.10097196

>>10093117
Try caffeine? If that doesn't work then maybe nicotine.

>> No.10097273
File: 16 KB, 639x369, 102P.png [View same] [iqdb] [saucenao] [google]
10097273

I got these parameters:
B=μnCoxW/L =2 mA/V , Vt= 0,7 V i λ = 0.05 /V, VDD=4 V i IR =50 μA
Find Io.

I have ZERO idea what I'm looking at, and prof is an asshole. Just please tell me what to Google and I'll try to figure out the answer myself, you don't have to literally spoon feed it to me.

>> No.10097306

>>10097273
Of course it's a fucking trick question, told ya prof is a pure asshole. Found the book he got this problem from, along with the solution. Fuck I hate this guy.

Io=Ir if opamps are ideal if anyone's wondering.

>> No.10097391
File: 120 KB, 1920x1080, cosine_problem.png [View same] [iqdb] [saucenao] [google]
10097391

I get that 1-cos(30) is used to calculate the height difference between G1 and G2, but how is it derived?

>> No.10097399

>>10097391
lol brainlet

>> No.10097400

>>10097399
Care to explain instead of just being a douche in the "stupid questions" thread?

>> No.10097402

>>10097051
Community college?

>> No.10097406

>>10097402
> 30 year old NEET virgin wizard in a community college.
I'd stand out like a sore thumb. I prefer reading alone and /sqt/

>> No.10097417

>>10097406
When I did my AS in Engineering Science, there were a substantial amount of people around your age.

Some were even older with graying hair. A lot of them were technicians looking to advance their careers.

>> No.10097420

>>10097391
It might have to do with the weird swing rope stretching that's going on. I guess they're saying the stretch to the different length is instantaneous.

In the 'first before 30' section take the length of the rope at theta = 0 to be 2.80 m. Next for the 'just after 0' section take the length of the rope at theta = 0 to be 3.00 m.

>> No.10097646

>>10096976
anyone?

>> No.10097648

>>10097051
Read Euler's Elements of Algebra

>> No.10097667
File: 27 KB, 277x337, gril.png [View same] [iqdb] [saucenao] [google]
10097667

>>10097391

The literal definition of cosine.

>> No.10097746
File: 2.47 MB, 4032x3024, D9D941A4-C10A-4743-A8B7-09EA402B608B.jpg [View same] [iqdb] [saucenao] [google]
10097746

Question on circuits.
So when this circuit is closed for a long time, thei inductor becomes short. So if I were to calculate the current, wouldn’t it just be the current through the 75k resistor? Why do I need to combine the 50k and 75k to make a 30k resistor? Since there’s a short, wouldn’t the current just ignore the 50k path?

>> No.10097799
File: 397 KB, 3024x4032, Untitled.png [View same] [iqdb] [saucenao] [google]
10097799

>>10097746
>implying we're going to read sideways for you

>> No.10097804

>>10097799
woah how'd you do that?

>> No.10097807

>>10097804
Print out. Hand erase gridlines. Scan. Easy.

>> No.10097811

>>10097746

You seem very confused, it is hard to understand what you mean. For this reason, my answers are guesses as to what the methodology you're supposed to use are.

You are right about the 50k resistor being irrelevant for the voltage-source provided current through the inductor.

> why [...] combine the 50k and 75k to make a 30k resistor
Because you need to know the current through the inductor before the switch is closed. Part of this current is provided by the current source to the left of the diagram, and from this point of view the total resistance is:
5k || (20k + 50k||75k).

You also need that resistance as a part of the calculation for tau.

>> No.10097827
File: 28 KB, 505x213, cir.png [View same] [iqdb] [saucenao] [google]
10097827

>>10097746
You dont

a)
75 = 75*i_2 + 50(i_2-i);
50(i-i_2) + 20*i + 5*(i+50) = 0

i=-4mA

b) i=1mA

c) T = L/R = 200/75 us

d) i(t)= i_b - (i_b-i_a)e^-t/T = 1-5e^-t/(2.666...us)

>> No.10097836

>>10097804
gimp
image -> transform -> rotate 90° clockwise
image -> mode -> grayscale
colors -> levels -> auto -> shift white down till gray disappears

>> No.10097856

>>10097804
>print image out
>shine bright lamp on printout
>take picture
>turn picture into black and white using method of choice
>rename image to cir.png
>solve machine learning labor task
>upload to 4chan

>> No.10097857

>>10097804
just train a neural network to do it

>> No.10097864

>>10097857
ok

>> No.10097866

>>10097864
thanks for your reply it means a lot to me

>> No.10097869

>>10097827
I tried this method, except I used source transformation on the left side. Doing that gave me 250V source and a 5k and 20k in series, but I get -1mA.
Is there something wrong with my calculation or am I not allowed to do that?

>> No.10097872

>>10097869
I also tried Node-Voltage and I still get 1mA but the answer should be -4mA

>> No.10098069

>>10091015
I have a potentiometer and a voltage source.... How would I get a linear relationship between V_out and V_in?

Right now, I'm thinking of using an opamp in a regular negative feedback configuration, but I'm not sure.

>> No.10098088

>>10098069
Put it up your butt

>> No.10098101

>>10096045
anyone got any ideas on good college level books on wildlife biology in my area?

>> No.10098116
File: 21 KB, 391x400, black programmer.jpg [View same] [iqdb] [saucenao] [google]
10098116

>>10097856
>>solve machine learning labor task

>>>/g/tfo

>> No.10098123

>>10097869
i*25+250 + (i-i_2)*50 = 0;
75 = 75*i_2 + 50(i_2-i);
https://www.wolframalpha.com/input/?i=solve+%7Bi_1*25%2B250+%2B+(i_1-i_2)*50+%3D+0,+75+%3D+75*i_2+%2B+50(i_2-i_1)%7D

still -4ma

>> No.10098233

Can anyone recommend textbooks on rocketry and orbital mechanics? Thank

>> No.10098238

>>10098088
Okay now what.

>> No.10098250

>>10091015
>be a faraday cage
>no electric fields within me unless originating from my cavity itself
>isolated from the outside noise
>somehow light still goes right through me

why can light penetrate faraday cages as its an EM wave?

>> No.10098327

>>10098233
https://4chan-science.wikia.com/wiki/Mechanical_and_Aerospace_Engineering#Orbital_Mechanics
https://4chan-science.wikia.com/wiki/Mechanical_and_Aerospace_Engineering#Propulsion

>> No.10098390

>>10097667
thanks for clearing that up

>> No.10098475

I need optical simulation software, but I don't have any money, what should I use?

>> No.10098477

>>10098475
kneepads

>> No.10098484

>>10098477
haha, very funny. While kneepads might have interesting properties in the microwave range and might actually act as lenses, I am trying to work in the visible light range.

>> No.10098541

What identities do I have to use to show this equals tan^3x=1.5

>> No.10098542
File: 3.12 MB, 4032x3024, 20181027_011251.jpg [View same] [iqdb] [saucenao] [google]
10098542

>>10098541
Forgot pic

>> No.10098555

>>10098541
>>10098542
all you need is sin / cos = tan, the rest is just algebra

fuck captcha

>> No.10098644

Let’s say we have twins that looked exactly alike. We separate them to two different families and they grow up to be yound adults. If we were to go back in time and switch them, would they lead exactly the same lives the other person had?

>> No.10098764

>>10098555
Lol I'm a brainlet

>> No.10098945

Why can't you grab random organic matter and covert it into something edible by throwing it in a big fermentation environment?

>> No.10099068
File: 3.76 MB, 4608x3456, 20181027_152812.jpg [View same] [iqdb] [saucenao] [google]
10099068

Can anyone help me with that one? Idk what to do.

>> No.10099144

>>10099068
https://en.wikipedia.org/wiki/Squeeze_theorem

>> No.10099175

>>10099068
Literally apply the definition of limit.

>> No.10099327
File: 205 KB, 1298x560, fuck.jpg [View same] [iqdb] [saucenao] [google]
10099327

can anybody help me with any (preferablly all desu) of these questions?
I know the equations for linearisations and differentials and all that stuff but I'm not sure what is supposed to go where, sorry for being ultra brainlet

>> No.10099347

I have my analysis I test in about 18 hours and I know jackshit of integration, and forgot whatever i learned about limits in my past exams
any resource I can frantically dive myself into to remedy or will I just have to wing it?

>> No.10099441

how does the internet work

>> No.10099450

>>10099441
computers

>> No.10099783

Here's a real life probability problem I'm having right now.

I have a test coming up, where I need to study everything about 132 different drugs. On the day of the test, I have to blindly draw 3 pieces of paper, each one with a different drug name. If I only studied 70 drugs, whats the probability of me drawing AT LEAST 1 drug that I have studied? How many drugs should I study to get a good chance of drawing at least 1 that I studied?

>> No.10099800

>>10099327
If D=4.7w^2/3, then solve for 1.03D=4.7x^2/3 and 0.97D=4.7x^2/3 and you have your bounds for the error.
Don't remember the meanings.
It just wants F'(35) and F'(55), if I ain't mistaken.

>> No.10099991

Is the lethality of rabies considered an "unusual" quality, or are there plenty of 100% fatality infectious diseases that are just more obscure?

>> No.10100027

>>10098123
This makes no sense. You're saying the current through the 50k resistor is (i-i_2) and (i_2-i)

>> No.10100111

>>10099783
Here's one way to do this analysis.

Say you have N total things to study you study x of them. First of we assume it doesn't matter what order the three things are given to you during the test. So there are C(N,3) possible test drug sets. Now [eqn]P(\text{drug set includes at lest 1 drug you studied}) = 1 - P(\text{drug set includes none you studied}).[/eqn] Well,[eqn] P(\text{drug set includes none you studied}) = \frac{\text{# drug sets including drugs not studied}}{\text{# drug sets possible}}.[/eqn] The # drug sets including drugs not studied is C(N-x,3). So from the above we find, [eqn]P(\text{test drug set includes at least 1 drug you studied}) = 1-\frac{C(N-x,3)}{C(N,3)} = 1-\frac{(N-x)(N-(x+1))(N-(x+2))}{N\left(N-1\right)\left(N-2\right)}.[/eqn] For x = 70 and N = 132 the probability is about 90%. You only need to study 27 for 50%.

>> No.10100115

>>10096777
>>10096923
How is that different from substitution though? Slightly taken out of context, but isn't it the same concept as this
φ'(u) = 2 · u
dx = φ'(u)du
or this?
u = ln(x) du = (1/x) dx (from when integrating ln(x))
What am I missing here?

>> No.10100120

Am I the only one for whom /sci/TeX isn't working? Could you help me fix this?

>> No.10100121

>>10100111
ugh, [eqn]P(\text{test drug set includes at least 1 drug you studied}) = 1-\frac{C(N-x,3)}{C(N,3)} = 1-\frac{(N-x)(N-(x+1))(N-(x+2))}{N(N-1)(N-2)}.[/eqn]

>> No.10100139

>>10100120
adblock

>> No.10100144

>>10091015
so like companies get paid to make hair thicker (actual hair diameter wider). how do you make hair thinner? it would be a gold mine because women cant go bald without paying for wigs and hair plugs ect

>> No.10100153

>>10100120
if you're using ublock,
@@||4chan.org^$csp
I wouldn't recommend it desu, hiroshima has been doing some bad shit lately and letting scripts from 4chan run unchecked is a bad idea nowadays

>> No.10100163

>>10096923
>"CA tutor"
>doesn't understand basic calculus
Holy shit lmfao

>> No.10100170

>>10096931
He's asking on what space [math]X[/math] do we have the notion of differentiation and integration WITHOUT reference to Euclidean space. You'd need MUCH more than completeness.
Didn't want to believe that you are actually this stupid but I guess I really have no choice now

>> No.10100179
File: 2 KB, 164x48, indicator.png [View same] [iqdb] [saucenao] [google]
10100179

Why is the indicator function a projection? Isn't idempotency of a function defined as the composition of itself instead of the square?

>> No.10100184

>>10100115
It's ultimately Radon-Nikodym. If you have a measures [math]\mu\ll\nu[/math] then a measurable Nikodym derivative [math]f[/math] exists such that over any Borel set [math]E[/math] you have [eqn]\nu(E) = \int_E fd\mu[/math]. Apply this to [math]\mu = \operatorname{id}_\mathbb{R}[/math] and [math]\nu = y[/math], then if [math]y(x)[/math] is an absolutely continuous function on [math]\mathbb{R}[/math] then the Radon-Nikodym derivative is just [math]y'[/math].

>> No.10100224

Have any discoveries been made by dissecting neural networks?

>> No.10100237

>>10100115
yes but what is dx or du? Sure, it works, but only because the symbols just happen to match up, but you are implicitly using the fundamental theorem of calculus. Now try it with a second derivative and tell me if it works (protip: it wont)

>> No.10100249

>>10100163
what's wrong huh?
>>10100170
you're the retard. He specifically mentions when can he do analysis over a field and mentions two things that arent fields. And sure, you can't do differentiation and integration with just completeness, but those aren't just the only two concepts of analysis...

>> No.10100266

>>10100249
in fact, you dont even need completeness for differentiation or integrability

>> No.10100291

>>10100266
>you don't need completeness for integration
Imagine I have a set S that I want to measure. So I pick a set that contains S and has some measure, then I pick a set that has a smaller measure than the previous one but still contains S, and so on. I can't show that this sequence of sets converges to some measure, so I can't say that there is a minimum measure for sets containing S unless I can explicitly present this set. So Riemann integration completely stops working 99% of the time, but you technically aren't really wrong.
>you don't need completeness for differentiation
That's wrong.

>> No.10100312

Can anything be done to beeswax to make it digestible by humans(as opposed to just acting like fiber)?

>> No.10100355

>>10100291
then how do non-complete smooth manifolds exist then?

>> No.10100380

>>10100355
Because the constant function is differentiable literally anywhere.

>> No.10100386

>>10100380
My bad, literally anywhere that has an additive identity.

>> No.10100388

>>10100386
Also, local completeness and general completeness aren't equivalent.

>> No.10100460

>>10100111
thanks homie

>> No.10100520

>>10091015
What is the minimum amount of episodes of Haruhi you need to watch in order to watch season 1 in every possible order?

>> No.10100560

What is the mathematical probability I will get a gf?

>> No.10100570

>>10100560
100%

>> No.10100577

>>10100560
0%

>> No.10100582

>>10100560
200%

>> No.10100868

I'm getting confused about something regarding conjugation in groups.
We know that a conjugation [math]\phi : G \longrightarrow G , \phi (x) = gxg^{-1}[/math] (where g is fixed) is a bijection, specificially an automorphism. However, I'm getting confused about the cardinal of the conjugacy class of x. If I defined another function (which I assume is not necessarily a group homomorphism anymore) [math] \phi_{2} : G \longrightarrow G , \phi_{2} (x) = xgx^{-1}[/math], then the conjugacy class is just the image of [math]\phi_{2}[/math] and has nothing to do with the other function [math] \phi[/math], right? I think I'm getting confused because the former is usually called a conjugation function and yet the conjugacy class of g isn't it's image. Is this why conjugacy classes aren't subgroups, then? Because they are images of functions that aren't necessarily group homomorphisms, unlike [math] \phi [/math]?

>> No.10100934

What does the trace of a matrix represents in geometric terms?

>> No.10100938

What's the best book for circuit analysis? I don't like any of the "classics", like Sadiku. They aren't rigorous enough for my taste.

>> No.10100942

>>10100934
>What does the trace of a matrix represents in geometric terms?
https://mathoverflow.net/questions/13526/geometric-interpretation-of-trace

>> No.10100943

>>10100577
>>10100560
Actually he is rigth beacuse, he doesnt have any of you data to start doing the calculation, also your are gonna die alone

>> No.10100988 [DELETED] 

>>10100027
2 have to loops that overlap at the 50K resistor, one going up and the other going down. By super position, the net current is their difference. Which difference is the direction you're going around in the loop.

>> No.10100994

>>10100027
You have two loops that overlap at the 50K resistor, one going up and the other going down. By super position, the net current is their difference. Which difference is the direction you're going around in the loop.

>> No.10101008

>>10100027
You have to loop that overlap at the 50K resistor, one going up and the other going down. By super position, the net current is their difference. Which difference is the direction you're going around in the loop.

>> No.10101058

https://en.wikipedia.org/wiki/PSO_J318.5-22

HOW THE FUCK CAN YOU EVEN DETECT A PLANET FLYING AROUND IN THE INTERSTELLAR VOID 80 FUCKING LIGHT YEARS AWAY

>> No.10101249

>>10094566
>>10092212
Burhs?

>> No.10101255

>>10101058

> We have never before seen an object free-floating in space that looks like this. It has all the characteristics of young planets found around other stars, but it is drifting out there all alone

What does it mean?

>> No.10101264

>>10092212
Watch Hamming's stuff.

>> No.10101270

>>10101264
Thanks, quick check and it looks very interesting.
Have a nice day/evening!

>> No.10101411

>>10100868
start thinking as conjugation of g more in terms of "what elements commute with g". If gxg^-1 = x, then gx=xg, that is, g commutes with x. Then, the stabilizer under conjugation by g, is the subgroup of precisely those elements that commute with g.

Also, conjugacy classes are just a fancy name for the distinct orbits under the conjugation action. In general, only one orbit is going to have the identity in it since orbits partition the group into distinct subsets.

In your function (which is not a group homomorphism since the identity is not sent to the identity), [math]\phi_2^{-1}(g)[/math] is precisely the stabilizer of g. Also it's not a well defined group action, so you can't really speak about its orbits etc.

>> No.10101594

What physically is energy? What does it look like? Is it made of atoms?

"The ability to do work" makes a circular definition.

>> No.10101607

>>10101594
It sort of has to be circular. Like how ypu can't say what is movement.

>> No.10101660

>>10101594
Every definition, axiom, logical solution, idea, everything *in* our universe in the end is circular because we cannot, due to being constrained TO THIS UNIVERSE, check what is necessary and what is not against alternative universe.

David Hume figured this out, you know!

>> No.10101661

Can someone guide me to a book that explains what 'space' is. And I don't just mean the Space as in Stars and Earth and shit. But what the 'space' is that they occupy. How things can have dimensions. What is the dimension and how the fuck it actualizes into space that thing can occupy

it has minducked me for 1 year now.
i simply don't get
what is the space things occupy and how it is there

>> No.10101671

>>10092317
what is your intuition?

>> No.10101679

Could it be that some (if not most) passable traps have Klinefelter syndrome?

>> No.10101705

>>10101679
By the way, this is a legitimate question of mine, born out of sheer curiousity.

>> No.10101707

>>10101661
Space is a locally 4 dimensional Riemannian manifold

>> No.10101709

>>10101707
bruh but
one step backwards

how did it fucking manifest
what space does the space itself occupy

>> No.10101797

>>10101679
No. Klinefelter is pretty recognizable, and most passable traps are short.

>> No.10101836

>>10091015
How do I know if a function is integrable?

>> No.10101838

>>10101836
>How do I know if a function is integrable?
Check whether the function satisfies the definition of integrable.

>> No.10101841

>>10101838
>the definition of integrable.
So it just has to be continuous?

>> No.10101850

>>10101841
>So it just has to be continuous?
If that's your definition of integrable, then yes.

>> No.10101902

>>10101836
Lebesgue criterion

>> No.10102001

>>10091015
What major should i choose if i have shit memory, can't learn (and if try and spend time on it i get even worse results than without it), hate and can't do programming (unless i copy whole lines of code in order for something to work), can't understand most of physics theoretical part, can't properly remember everything that basics in chemistry demands and I'm only able to calculate fast as far as my math skills go?

>> No.10102016

>>10102001
Business

>> No.10102026

>>10102016
>business
yeah, i thought about taking that one, but once i started reading economic materials and noticed how much emotionally focused instead of a effeciency focus they've had i got bored and dropped that idea

>> No.10102030

>>10101709
when did you start being a faggot? unanswerable

>> No.10102031

>>10102030
AH BUT SPACE

IS SPACE IS SPACE.
BUT THE SPACE THE NARROW SPACE OCCUPIES

WHAT IS IT
HOW CAN BE A THING

?!

>> No.10102040
File: 5 KB, 290x290, example.png [View same] [iqdb] [saucenao] [google]
10102040

I have an exercise on my hands asking me to match diffraction gratings of various shapes to their Fraunhofer diffraction pattern (eg: 2D Fourier Transform), but I have no idea how to do this without going on MATLAB and calculating them directly.

How do I figure out by heart what this thing's FT is going to be?

>> No.10102233
File: 38 KB, 1308x381, Capture.png [View same] [iqdb] [saucenao] [google]
10102233

>>10091015
Where am I going wrong here? I've been multiplying latent heat of fusion with mass though clearly receive the wrong answers.

Cute image by the way, given your encourage I won't sleep tonight (not that I was going to either way), bless she looks so overwhelmed by a small dosage of caffeine.

>> No.10102250

>>10102233
The heat for fusion is transferred at 0°C
You first have to transfer enough heat to heat the ice from -5 to 0°c, did you keep that in mind?

>> No.10102306
File: 31 KB, 960x540, Surprised.jpg [View same] [iqdb] [saucenao] [google]
10102306

>>10102250
My brains not working here. I considered the change in heat though tried subtracting the 5 degrees from the fusion heat, that didn't work. The annoying thing is that this is going to be so obvious when I wrap my head around it but for God knows why isn't making sense at the moment. Let's see
>ice needs heat to transfer into liquid
>heat is transferred at 0 degrees
>ice needs to be heated from 5 degrees to 0 so it melts albeit very slowly
God fucking damn it, went through every equation on the past two powerpoints and they aren't doing anything.
Used specific heat formula;
Q = M x CP x ΔΘ
Then inputted values for Q
>Q = 19.9 x 2108 x 5
>got completely wrong answer

Used
>Q = M x λ (subscrip f)
Inputted values pretending that Q wasn't 5 because then it'd solve itself
>Q = 19.9 x 334
>again completely wrong

I don't get it, you have to transfer heat so the the overall heat increases, where this heat comes from fuck knows but it works (2108J of heat on ice would presumably be roasting in the first place) and if water is fusing like a pack of yu-gi-ohs at over 334 kJ its probably magma. Still, we need heat to heat the heat so stuff melts, we need to know how much heat is inputted in this case, initially the ice is at 5 degrees then goes to 0 degrees, specific heat then means nothing so it might as well not be there, fuck off specific heat and drink bleach. What do we use then?

I hate it when this happens, its the only thing that annoys me - failing at the most basic shit that 10 year olds can do, might as well be dead if that's the best I can do.

>> No.10102374

>>10091015
Can someone give me a sanity check? Its been ages since I've done statistics and I'm ashed to say I've forgotten a lot of it. I've managed to simplify the core idea of the problem I'm solving down to this analogy.

Suppose you have two bags. Each bag contains two balls. In one of the bags both of the balls is black. In the other one is black and the other is red.

Suppose you pick up a bag and pull out a ball. It's black. Is it more likely that the red ball is in the other bag?

My thinking is yes. The probability that you picked the right bag and pulled a black ball is 1/2 * 1/2 = 1/4. The probability that you picked the wrong bag and pulled a black ball is a 1/2. And for completeness sake, the event that didn't happen given, the probability that you picked the right bag and pulled a red ball is a 1/4.
Since we know this last event didn't occur then in two out of the three possible events that did occur the red ball is in the other bag.

I'll muddled my way through solving the bigger problem. Right now I'm just making sure I understand the fundamentals.

>> No.10102489

>>10100237
What makes it okay for this "multiply by dx" technique to be employed in integration by parts/substitution then? I still haven't really understood what makes it acceptable or not. I mean, I'm aware that d/dx is essentially just an operator but this kind of solution to an integration problem seems to come up all the time. Unless you are arguing that it generally should not be employed at all?

>> No.10102493

I want an equation for a "proportional" controller where the bounded is restricted to [0,1]. I'm ok with it not being strictly proportional, as, for example, if the process value is way too high and the set point is low, the set point shouldn't decrease by as much as it would when the set point is high. I can't figure out how to put it into a form that works.

https://en.wikipedia.org/wiki/Proportional_control

>> No.10102500

>>10102489
Multiplying by dx is not well-defined. However, "multiplying by dx then putting an integral sign" is well-defined. So just do that

>> No.10102510

>>10102493
setpoint is restricted*

>> No.10102619

Stupid genetics question google can't answer for me: let's say I have the DNA strand AGGATTACGATCGCACAAGATCCT
Which translates as RNA to UCCUAAUGC UAGCGUGUUCUAGGA
Then the protein sequence starts at AUG and should stop at UAG. But if I do that, there's a pair number of bases in between them, which means there stop codon is shifted or something. Wtf do I do? does it not stop at UAG? or does it just somehow not translate the UC before UAG into proteins?

>> No.10102620

>>10102619
>UCCUAAUGC UAGCGUGUUCUAGGA
I didn't mean to put a space there, sorry if it's confusing. That has nothing to do with the stop codon I mentioned, which is the very last UAG.

>> No.10102629

>>10102628
new
>>10102628